LSAT and Law School Admissions Forum

Get expert LSAT preparation and law school admissions advice from PowerScore Test Preparation.

User avatar
 Dave Killoran
PowerScore Staff
  • PowerScore Staff
  • Posts: 5853
  • Joined: Mar 25, 2011
|
#88621
Complete Question Explanation
(The complete setup for this game can be found here: lsat/viewtopic.php?f=225&t=35163)

The correct answer choice is (C)

This is a harder-than-average List question. Our analysis of who is first and last only eliminates answer choice (D)—either L or O must be last, yet (D) features L in seat 3 and O in seat 5.

Answer choice (A): If V is in seat 5, then O must be in seat 6 since V O. However, this answer features V in seat 5 and L in seat 6, and so it is incorrect.

Answer choice (B): There are two different ways this answer can be eliminated:

1. If S rows in seat 3, then M and Z must occupy seats 1 and 2 in some order. However, this answer features Z in seat 5, and that assignment does not leave enough athletes for seats 1 and 2.
2. According to the rules L must row behind S, and O must row behind V and Z. With S, V, and Z rowing at seats 3, 4, and 5, only seat 6 is available for both L and O, a violation of the rules.
Answer choice (C): This is the correct answer choice.

Answer choice (D): This answer choice is incorrect because either L or O must be in seat 6, and neither can be under this scenario.

Answer choice (E): According to the rules, M and S must row ahead of V. When Z and V row at seats 2 and 3, only seat 1 is available for both M and S, a violation of the rules.

Get the most out of your LSAT Prep Plus subscription.

Analyze and track your performance with our Testing and Analytics Package.